Publicaciones Recientes

Problema

Grado de repulsión de una función circular

Enviado por jmd el 10 de Diciembre de 2011 - 15:44.

Una función $f: N \mapsto N$ es circular si para cada $p$ en $N$ existe $n$ en $N$ con $n\leq p$ tal que:
$$\underbrace{f^n(p) = f(f(\ldots f(p) \ldots )))}_{n veces}=p$$
La función $f$ tiene grado de repulsión $k$, $0 < k < 1$, si para cada $p$ en $N$, $f^i(p) \neq p$ para $i\leq [k\cdot p]$. Determine el mayor grado de repulsión que puede tener una función circular. Nota: $[x]$ indica el mayor entero menor o igual que $x$.

 

Problema

... y se forma un trapecio isósceles...

Enviado por jmd el 10 de Diciembre de 2011 - 15:38.

La circunferencia inscrita en el triángulo $ABC$ es tangente a $BC, CA$ y $AB$ en $D, E$ y $F$, respectivamente. Suponga que dicha circunferencia corta de nuevo a $AD$ en su punto medio $X$, es decir, $AX = XD$. Las rectas $XB$ y $XC$ cortan de nuevo a la circunferencia inscrita en $Y$ y en $Z$, respectivamente. Demuestre que $EY = FZ$.

Problema

Dominio eficiente de un tablero

Enviado por jmd el 10 de Diciembre de 2011 - 15:36.

En un tablero de $m\times m$ casillas se colocan fichas. Cada ficha colocada en el tablero "domina" todas las casillas de la fila (--), la columna (|) y la diagonal (\), a la que pertenece. Determine el menor número de fichas que deben colocarse para que queden "dominadas" todas las casillas del tablero. Nota: la ficha no "domina" la diagonal (/).

Problema

Perpendicular común a dos rectas en el espacio

Enviado por jmd el 10 de Diciembre de 2011 - 15:34.

Sean $r$ y $s$ dos rectas ortogonales y que no están en el mismo plano. Sea $AB$ su perpendicular común, donde $A$ pertenece a $r$ y $B$ a $s$. Se considera la esfera de diámetro $AB$. Los puntos $M$, de la recta $r$ y $N$, de la recta $s$, son variables, con la condición de que $MN$ sea tangente a la esfera en un punto $T$. Determine el lugar geométrico de $T$. Nota: el plano que contiene a $B$ y $r$ es perpendicular a $s$.

Problema

Condiciones extravagantes para n+1 números

Enviado por jmd el 10 de Diciembre de 2011 - 15:32.

Sea $n$ un número entero mayor que 1. Determine los números reales $x_1, x_2,\ldots, x_n\leq 1$ y $x_{n+1}>0$, que verifiquen las dos condiciones siguientes:
$$\sqrt{x_1}+\sqrt[3]{x_2}+\ldots+\sqrt[n-1]{x_n}=n\sqrt[2]{x_{n+1}}$$
$$\frac{x_1+x_2+ \ldots +x_n}{n}=x_{n+1}$$

Problema

Para entender la pregunta primero tienes que responderla

Enviado por jmd el 10 de Diciembre de 2011 - 15:27.

Determine los posibles valores de la suma de los digitos de todos los cuadrados perfectos.

Problema

Si le entiendes al enunciado obtienes un punto

Enviado por jmd el 10 de Diciembre de 2011 - 14:20.

Demostrar que todo número natural $n\leq 2^{1000000}$ puede ser obtenido a partir de 1 haciendo menos de 1100000 sumas; más precisamente: que hay una sucesión finita de números naturales $x_0, x_1,\ldots,x_k$, con $k < 1100000$, $x_0 = 1, x_k = n$ tal que para cada $i = 1, 2,\ldots, k$, existen $r, s$ con $0\leq r < i, 0 \leq s < i$, y $x_i = x_r + x_s$.

Problema

Eliges, sumas, y te vas...

Enviado por jmd el 10 de Diciembre de 2011 - 14:18.

Sean $n, r$ dos enteros positivos. Se desea construir $r$ subconjuntos $A_1, A_2,\ldots, A_r$ de $\{0, 1,\ldots, n-1\}$ cada uno de ellos con exactamente $k$ elementos y tales que, para cada entero $x$, $0\leq x \leq n-1$, existen $x_1$ en $A_1$, $x_2$ en $A_2$ ,... , $x_r$ en $A_r$ (un elemento en cada conjunto) con $x = x_1 + x_2\dots+ x_r$. Hallar el menor valor posible de $k$ en función de $n$ y $r$.

Problema

Transformación de acutángulo a equilátero (en el circuncírculo de aquél)

Enviado por jmd el 10 de Diciembre de 2011 - 14:16.

Se dan los puntos $A, B, C$ sobre una circunferencia $K$ de manera que el triángulo $ABC$ sea acutángulo. Sea $P$ un punto interior a $K$. Se trazan las rectas $AP, BP, CP$, que cortan de nuevo a la circunferencia en $X, Y, Z$. Determinar el punto $P$ que hace equilátero al triángulo $XYZ$.

Problema

Tablero lampareado

Enviado por jmd el 10 de Diciembre de 2011 - 14:14.

En cada casilla de un tablero $n\times n$ hay una lámpara. Al ser tocada una lámpara, cambian de estado ella misma y todas las lámparas situadas en la fila y la columna que ella determina (las que están encendidas se apagan y las apagadas se encienden). Inicialmente todas están apagadas. Demostrar que siempre es posible, con una sucesión adecuada de toques, lograr que todo el tablero quede encendido y encontrar, en función de $n$, el número mínimo de toques para que se enciendan todas las lámparas.

Distribuir contenido